Akademisyenler öncülüğünde matematik/fizik/bilgisayar bilimleri soru cevap platformu

Answers posted by Dogukan633

70
answers
18
best answers
2 votes
cevaplandı 11 Eylül 2018
daha genel halde,                                                              $(n^a - 1,n
1 vote
cevaplandı 22 Ağustos 2018
$n>1$ tamsayısının bütün pozitif bölenlerini bir $S$ kümesinde toplayalım. $S$ kümesi iyi sıral
0 votes
cevaplandı 16 Ağustos 2018
Yanıt : Hayır Her $p$ asalı için                                                           
1 vote
cevaplandı 5 Haziran 2018
Herhangi bir $m$ bileşik sayısı için de devirlidir. Ve $p$ wieferich asalı olmayan bir asal sayı
0 votes
cevaplandı 28 Nisan 2018
hiçbir $n$ için bölmesin. Bu durumda                                                           
0 votes
cevaplandı 5 Şubat 2018
                                     $111111...1 = \dfrac {1} {9}  .(10^{3^{n}} - 1)$ $LTE
1 vote
cevaplandı 3 Şubat 2018
Carmichael'in lambda fonksiyonuna göre, eğer  $ 8 | m $ ve $(n,8)=1$ ise                  
0 votes
cevaplandı 23 Aralık 2017
Aritmetik ortalama büyük eşit geometrik ortalama uygulanırsa, eşitlik durumunun sadece $x^3 = y^3
1 vote
cevaplandı 21 Ekim 2017
Aslında aklına şu sorular gelmesi lazım.                                                  
0 votes
2 votes
cevaplandı 6 Ekim 2017
$\dfrac {a} {b} = \dfrac {1} {1}+ \dfrac {1} {2} + ... + \dfrac {1} {p-1}$ toplamını $(\dfrac {1}
2 votes
cevaplandı 25 Ağustos 2017
Kenar uzunlukları sırasıyla $ 5,8,7$ olan bir $ABC$ üçgeni alalım. Bu ABC üçgeninin içinde, $ m(A
0 votes
cevaplandı 9 Nisan 2017
En büyük çarpanı aradığımızdan toplanan sayılar arasında $ 4$  olamaz. Misal $ 5 = 4+1$  yazmak ye
2 votes
cevaplandı 14 Mart 2017
B4851 $ \dfrac {1} {x_{1}}+\dfrac {1} {x_{2}}+\dfrac {1} {x_{3}}\leq \dfrac {p^{2}} {3r}$ o...
0 votes
cevaplandı 19 Şubat 2017
$8 = 7+1$ $6$ = $7-1$ şeklinde yazılabilir. Bu durumda $(7+1)^{63}  + (7-1)^{83}$ i
2 votes
cevaplandı 14 Ocak 2017
$n-1\choose r-1$ + $n-1\choose r$ ifadesini açarsak $\dfrac {\left( n-1\right) !} {\left( n-r\r
1 vote
cevaplandı 13 Ocak 2017
Parçalanış sayısı ile alakalı  $P\left( n\right) \approx \dfrac {e^{\pi \sqrt {\dfrac {2n} {3}}
0 votes
cevaplandı 13 Ocak 2017
$r_{1},r_{2},\ldots r_{\varphi \left( n\right) } $ kalanları $Mod$ $n$ de elde edilebilecek, $n$
1 vote
cevaplandı 30 Aralık 2016
$OA = cosa$ ve $AP = sina$ olacağı çok açıktır. Aynı şekilde $OB = 1$ olduğundan $CB = tana$ olaca
20,200 soru
21,728 cevap
73,275 yorum
1,887,976 kullanıcı